Summe von n Quadraten (x21+x22+⋯+x2n)2(y21+y22+⋯+y2n)=z21+z22+⋯+z2n(x12+x22+⋯+xn2)2(y12+y22+⋯+yn2)=z12+z22+⋯ +zn2(x_1^2+x_2^2 + \dots + x_n^2)^2 (y_1^2+y_2^2 + \dots + y_n^2) = z_1^2+z_2^2 + \dots + z_n^ 2

Betrachten Sie diese 5-Quadrat-Identität,

( X 1 2 + X 2 2 + X 3 2 + X 4 2 + X 5 2 ) 2 ( j 1 2 + j 2 2 + j 3 2 + j 4 2 + j 5 2 ) = z 1 2 + z 2 2 + z 3 2 + z 4 2 + z 5 2

Wo,

z 1 = ( X 1 2 + X 2 2 + X 3 2 + X 4 2 + X 5 2 ) j 1 2 X 1 ( 0 X 1 j 1 + X 2 j 2 + X 3 j 3 + X 4 j 4 + X 5 j 5 ) z 2 = ( X 1 2 X 2 2 + X 3 2 + X 4 2 + X 5 2 ) j 2 2 X 2 ( X 1 j 1 + 0 X 2 j 2 + X 3 j 3 + X 4 j 4 + X 5 j 5 ) z 3 = ( X 1 2 + X 2 2 X 3 2 + X 4 2 + X 5 2 ) j 3 2 X 3 ( X 1 j 1 + X 2 j 2 + 0 X 3 j 3 + X 4 j 4 + X 5 j 5 ) z 4 = ( X 1 2 + X 2 2 + X 3 2 X 4 2 + X 5 2 ) j 4 2 X 4 ( X 1 j 1 + X 2 j 2 + X 3 j 3 + 0 X 4 j 4 + X 5 j 5 ) z 5 = ( X 1 2 + X 2 2 + X 3 2 + X 4 2 X 5 2 ) j 5 2 X 5 ( X 1 j 1 + X 2 j 2 + X 3 j 3 + X 4 j 4 + 0 X 5 j 5 )

Das Muster ist leicht zu erkennen,

( X 1 2 + X 2 2 + + X N 2 ) 2 ( j 1 2 + j 2 2 + + j N 2 ) = z 1 2 + z 2 2 + + z N 2

Der Fall n = 4 wird in Pfisters 8-Quadrat-Identität verwendet . Wie kann man beweisen, dass das Muster tatsächlich für ALLE positiven ganzen Zahlen n gilt ?

Gibt es ein Quadrat falsch auf X 1 2 + + X 5 2 im Titel und der ersten Gleichung?
Betrachten Sie den ersten Begriff. Erhalten ( X ich 2 ) 2 X 1 2 . Auch haben im zweiten Summanden des ersten Terms, 2 X 1 ( ( X ich j ich ) X 1 j 1 ) . Aber trotzdem unangenehm!
@emiliocba: Nein, ich habe es mit Mathematica überprüft und die 5-Quadrat-Identität gilt.

Antworten (1)

Wir können schreiben

z k = j k ( ich X ich 2 2 X k 2 ) 2 X k ich k X ich j ich = j k ich X ich 2 2 X k ( ich k X ich j ich + X k j k ) = j k ich X ich 2 2 X k ich X ich j ich
also hüpfen wir in einem kommutativen Ring
z k 2 = j k 2 ( ich X ich 2 ) 2 4 X k j k ( ich X ich 2 ) ( ich X ich j ich ) + 4 X k 2 ( ich X ich j ich ) 2
und schlussendlich
k = 1 N z k 2 = k = 1 N j k 2 ( ich X ich 2 ) 2 4 X k j k ( ich X ich 2 ) ( ich X ich j ich ) + 4 X k 2 ( ich X ich j ich ) 2 = ( ich X ich 2 ) 2 k = 1 N j k 2 4 ( k X k j k ) ( ich X ich 2 ) ( ich X ich j ich ) + 4 ( k X k 2 ) ( ich X ich j ich ) 2 = ( ich X ich 2 ) 2 k = 1 N j k 2 .

Stimmt es streng genommen:
k = 1 N 4 ( k X k j k ) ( ich X ich 2 ) ( ich X ich j ich ) + 4 ( k X k 2 ) ( ich X ich j ich ) 2 = 0
??????? Ich sage nicht, dass Sie falsch liegen, ich sage nur, dass es für mich bizarr aussieht. Haben Sie eine Möglichkeit, dies zu klären? Mein Problem liegt in der Tatsache, dass Sie Summierungen stornieren, die ähnliche Begriffe, aber unterschiedliche Indizes haben.
@ user22144: In der Gleichheit, die du schreibst, sollte es keine geben k = 1 N . Der letzte Schritt meiner Berechnung ist tatsächlich eine Folge davon, dass ich = 1 N C ich = J = 1 N C J .
Lieber @David: Vielen Dank! Als ich im Zusammenhang mit "Pfisters 8-Quadrat-Identität" zum ersten Mal auf die allgemeine Form stieß, nahm ich an, dass sie nur für n = 2^m gilt. Aber ein wenig Experimentieren mit Mathematica zeigte, dass es mehr als das war. Es ist gut zu wissen, dass es tatsächlich für alle n gilt. Danke noch einmal.
@DavidGiraudo, mein Fehler! Ich hatte die Summe falsch interpretiert. Kein Wunder, dass es so bizarr aussah.
Davide, vielleicht haben Sie Interesse daran: math.stackexchange.com/questions/751167/… :0)